LSAT 1 – Section 3 – Question 15

You need a full course to see this video. Enroll now and get started in less than a minute.

Target time: 1:29

This is question data from the 7Sage LSAT Scorer. You can score your LSATs, track your results, and analyze your performance with pretty charts and vital statistics - all with a Free Account ← sign up in less than 10 seconds

Question
QuickView
Type Tags Answer
Choices
Curve Question
Difficulty
Psg/Game/S
Difficulty
Explanation
PT1 S3 Q15
+LR
Strengthen +Streng
A
6%
147
B
73%
161
C
11%
155
D
2%
142
E
8%
152
133
146
158
+Medium 148.102 +SubsectionMedium
This page shows a recording of a live class. We're working hard to create our standard, concise explanation videos for the questions in this PrepTest. Thank you for your patience!

This is a strengthening question, as the stem asks us: Which one of the following, if true, supports the conclusion in the passage?

This is one of those questions where you can’t help but let out a sigh when you turn the page on your LSAT and it greets you. A long stimulus doesn’t necessarily mean a difficult question, but it usually means a time consuming one. We should always focus on sifting through the unnecessary details, and try to get a good grasp on the actual argument. This stimulus begins with a lot of context about computer software and how its standardization can make it vulnerable to viruses. Because computer software has become standardized, such that a business might have all its computers running on the same or similar software, a virus which infects one computer will have a high likelihood of being able to spread throughout the network to the other similar computers, giving a vandal the potential to destroy data on all the computers. Seems like a big problem! Luckily, our author has a solution; just make the software differ slightly between computers. By introducing minor variations that are compatible with business, the possibility of a virus destroying all data can be eliminate. This solution also wouldn’t entail any loss in computer compatibility. Our author concludes that this minor variation should be adopted. Our job is to strengthen this conclusion. Let’s see what the answer choices have in store for us:

Answer Choice (A) Interesting, but we’ve been explicitly told introducing minor variations won’t negatively impact this compatibility.

Correct Answer Choice (B) This strengthens our argument, since if true it would mean that whatever the costs associated with introducing minor variation are, they are worth avoiding the damage.

Answer Choice (C) Again, the author’s solution will not impact compatibility.

Answer Choice (D) Our conclusion is that a certain strategy will prevent a particular problem. The potential existence of other problems does nothing to strengthen the proposal.

Answer Choice (E) Interesting, but I don’t see how this would strengthen our proposed solution.

Take PrepTest

Review Results

Leave a Reply